matheraum.de
Raum für Mathematik
Offene Informations- und Nachhilfegemeinschaft

Für Schüler, Studenten, Lehrer, Mathematik-Interessierte.
Hallo Gast!einloggen | registrieren ]
Startseite · Forum · Wissen · Kurse · Mitglieder · Team · Impressum
Forenbaum
^ Forenbaum
Status Hochschulmathe
  Status Uni-Analysis
    Status Reelle Analysis
    Status UKomplx
    Status Uni-Kompl. Analysis
    Status Differentialgl.
    Status Maß/Integrat-Theorie
    Status Funktionalanalysis
    Status Transformationen
    Status UAnaSon
  Status Uni-Lin. Algebra
    Status Abbildungen
    Status ULinAGS
    Status Matrizen
    Status Determinanten
    Status Eigenwerte
    Status Skalarprodukte
    Status Moduln/Vektorraum
    Status Sonstiges
  Status Algebra+Zahlentheo.
    Status Algebra
    Status Zahlentheorie
  Status Diskrete Mathematik
    Status Diskrete Optimierung
    Status Graphentheorie
    Status Operations Research
    Status Relationen
  Status Fachdidaktik
  Status Finanz+Versicherung
    Status Uni-Finanzmathematik
    Status Uni-Versicherungsmat
  Status Logik+Mengenlehre
    Status Logik
    Status Mengenlehre
  Status Numerik
    Status Lin. Gleich.-systeme
    Status Nichtlineare Gleich.
    Status Interpol.+Approx.
    Status Integr.+Differenz.
    Status Eigenwertprobleme
    Status DGL
  Status Uni-Stochastik
    Status Kombinatorik
    Status math. Statistik
    Status Statistik (Anwend.)
    Status stoch. Analysis
    Status stoch. Prozesse
    Status Wahrscheinlichkeitstheorie
  Status Topologie+Geometrie
  Status Uni-Sonstiges

Gezeigt werden alle Foren bis zur Tiefe 2

Navigation
 Startseite...
 Neuerdings beta neu
 Forum...
 vorwissen...
 vorkurse...
 Werkzeuge...
 Nachhilfevermittlung beta...
 Online-Spiele beta
 Suchen
 Verein...
 Impressum
Das Projekt
Server und Internetanbindung werden durch Spenden finanziert.
Organisiert wird das Projekt von unserem Koordinatorenteam.
Hunderte Mitglieder helfen ehrenamtlich in unseren moderierten Foren.
Anbieter der Seite ist der gemeinnützige Verein "Vorhilfe.de e.V.".
Partnerseiten
Weitere Fächer:

Open Source FunktionenplotterFunkyPlot: Kostenloser und quelloffener Funktionenplotter für Linux und andere Betriebssysteme
StartseiteMatheForenWahrscheinlichkeitstheorieFast sichere Konvergenz
Foren für weitere Schulfächer findest Du auf www.vorhilfe.de z.B. Deutsch • Englisch • Französisch • Latein • Spanisch • Russisch • Griechisch
Forum "Wahrscheinlichkeitstheorie" - Fast sichere Konvergenz
Fast sichere Konvergenz < Wahrscheinlichkeitstheorie < Stochastik < Hochschule < Mathe < Vorhilfe
Ansicht: [ geschachtelt ] | ^ Forum "Wahrscheinlichkeitstheorie"  | ^^ Alle Foren  | ^ Forenbaum  | Materialien

Fast sichere Konvergenz: Frage (beantwortet)
Status: (Frage) beantwortet Status 
Datum: 12:54 Sa 18.06.2016
Autor: Septime

Aufgabe
Seien [mm] X_{1},X_{2},... [/mm] u.i.v. auf [0,1] gleichverteilte Zufallsvariablen. Zeigen Sie
[mm] \limes_{n\rightarrow\infty} (X_{1}*X_{2}*...*X_{n})^{1/n}=c\in\IR [/mm] fast sicher und bestimmen sie c.

Hallo,

mein Ansatz :

nun ist die Aussage [mm] X_{n} \to [/mm] X fast sicher äquivalent zu der Aussage, dass [mm] P(w\in O|\limes_{n\rightarrow\infty} (X_{1}(w)*X_{2}(w)*...*X_{n}(w))^{1/n}=c)=1 [/mm] ist, wobei O die Grundmenge vom Maßraum ist.
Da jede Funktion [mm] X_{i} [/mm] wegen Gleichverteilung auf [0,1] folgendermaßen aussieht

[mm] F(w)=\begin{cases} 0, & \mbox{für } w\le0 \mbox{ } \\ w, & \mbox{für } 0
gilt dann

[mm] F^{n}(w)=\begin{cases} 0, & \mbox{für } w\le0 \mbox{ } \\ w^{n}, & \mbox{für } 0
und damit

[mm] \limes_{n\rightarrow\infty} (X_{1}*X_{2}*...*X_{n})^{1/n}=\limes_{n\rightarrow\infty} (w^{n})^{1/n}=w [/mm]

Habe ich mir das zu einfach gemacht?

Ich freue mich auf jede Antwort.

Gruß Septime



Ich habe diese Frage in keinem Forum auf anderen Internetseiten gestellt.

        
Bezug
Fast sichere Konvergenz: Antwort
Status: (Antwort) fertig Status 
Datum: 17:59 Sa 18.06.2016
Autor: Gonozal_IX

Hiho,

> Da jede Funktion [mm]X_{i}[/mm] wegen Gleichverteilung auf [0,1]
> folgendermaßen aussieht
>  
> [mm]F(w)=\begin{cases} 0, & \mbox{für } w\le0 \mbox{ } \\ w, & \mbox{für } 0
>  
> gilt dann
>  
> [mm]F^{n}(w)=\begin{cases} 0, & \mbox{für } w\le0 \mbox{ } \\ w^{n}, & \mbox{für } 0

Jo.

> und damit
>  
> [mm]\limes_{n\rightarrow\infty} (X_{1}*X_{2}*...*X_{n})^{1/n}=\limes_{n\rightarrow\infty} (w^{n})^{1/n}=w[/mm]

Wie kommst du denn darauf? Dort steht doch das Produkt der Zufallsvariablen und nicht der Verteilungsfunktionen! Wo kommt denn dein [mm] $\omega$ [/mm] plötzlich her?

> Habe ich mir das zu einfach gemacht?

Du hast es schlichtweg falsch gemacht.
Tipp: Mache aus dem Produkt der Zufallsvariablen durch Anwendung einer geeigneten Funktion eine Summe und wende dann das starke Gesetz der großen Zahlen an.

Gruß,
Gono

Bezug
                
Bezug
Fast sichere Konvergenz: Frage (beantwortet)
Status: (Frage) beantwortet Status 
Datum: 06:12 So 19.06.2016
Autor: Septime

Ich habe versucht dein Tipp umzusetzen:

Es gilt
[mm] F_{X_{1}}=F_{X_{1}}=...=F_{X_{n}} \gdw \gamma_{X_{1}}=\gamma_{X_{2}}=...=\gamma_{X_{n}}, [/mm]
wobei [mm] \gamma_{X_{i}} [/mm] die charakteristische Funktion von [mm] {X_{i}} [/mm] ist.

Wegen Unabhängigkeit der [mm] {X_{i}} [/mm] folgt dann für alle [mm] t\in\IR [/mm]
[mm] \gamma_{X_{1}}(t)*\gamma_{X_{2}}(t)*...*\gamma_{X_{n}(t)} [/mm] = [mm] \gamma_{X_{1}+X_{2}+...+X_{n}}(t) [/mm] = [mm] E(e^{it(X_{1}+X_{2}+...+X_{n})}) [/mm]

und da die Verteilungsfunktionen von allen [mm] X_{i} [/mm] absolut stetig sind, ist die Verteilungsfunktion von [mm] X_{1}+X_{2}+...+X_{n} [/mm] ebenfalls absolut stetig und es folgt
[mm] E(e^{it(X_{1}+X_{2}+...+X_{n})})=\integral_{-\infty}^{\infty}{f_{ X_{1}+X_{2}+...+X_{n}}(x)*e^{itx}} dx=\integral_{-\infty}^{\infty}{P( X_{1}+X_{2}+...+X_{n}\le x)*e^{itx}} [/mm] dx

Nun weiß ich nicht mehr weiter. Mein Problem ist, dass ich nicht weiß wie ich das starke Gesetz der großen Zahlen sinnvoll benutzen soll. Zudem weiß ich nicht wie man von der Verteilungsfunktion oder dem Wahrscheinlichkeitsmaß zu den Zufallsvariablen kommt.

Gruß Septime


Bezug
                        
Bezug
Fast sichere Konvergenz: Antwort
Status: (Antwort) fertig Status 
Datum: 10:48 So 19.06.2016
Autor: Gonozal_IX

Hiho,

> Es gilt
>  [mm]F_{X_{1}}=F_{X_{1}}=...=F_{X_{n}} \gdw \gamma_{X_{1}}=\gamma_{X_{2}}=...=\gamma_{X_{n}},[/mm]
>  
> wobei [mm]\gamma_{X_{i}}[/mm] die charakteristische Funktion von
> [mm]{X_{i}}[/mm] ist.

das stimmt erst mal… ist aber absolut nicht zielführend.
Ich sehe auch nicht, wo du aus [mm] $(X_1\cdot\ldots\cdot X_n)$ [/mm] eine Summe gemacht hast… also noch deutlicher:
Verwende den Logarithmus und betrachte:

[mm] $Y_n [/mm] = [mm] \ln\left((X_1\cdot\ldots\cdot X_n )^{\frac{1}{n}}\right)$ [/mm]

1.) Forme [mm] $Y_n$ [/mm]  geeignet um, um das starke Gesetz der großen Zahlen anwenden zu können.

2.) Berechne damit [mm] $\lim_{n\to\infty} Y_n$ [/mm]

3.) Aus [mm] $e^{Y_n} [/mm] = [mm] (X_1\cdot\ldots\cdot X_n )^{\frac{1}{n}}$ [/mm] folgt dann die gewünschte Aussage.

Gruß,
Gono

Bezug
                                
Bezug
Fast sichere Konvergenz: Frage (beantwortet)
Status: (Frage) beantwortet Status 
Datum: 14:58 So 19.06.2016
Autor: Septime

Vielen Dank! Ich habe den Trick mit dem Logarithmus vorher noch nicht gekannt

Stimmt das jetzt ?

Es gilt
$ [mm] Y_n [/mm] = [mm] \ln\left((X_1\cdot\ldots\cdot X_n )^{\frac{1}{n}}\right) [/mm] $ = [mm] \bruch{1}{n}*ln(X_{1}*X_{2}*...*X_{n}) [/mm] = [mm] \bruch{1}{n}\summe_{i=1}^{n}ln(X_{i}) [/mm]
und mit dem starken Satz der großen Zahlen folgt
[mm] \bruch{1}{n}\summe_{i=1}^{n}ln(X_{i}) \to E(ln(X_{1})) [/mm] fast sicher.
Außerdem ist [mm] X_{1} [/mm] gleichverteilt, daher gilt für [mm] b\in(0,1] [/mm]
[mm] E(ln(X_{1}))=\limes_{b\rightarrow0}\bruch{ln(1)+ln(b)}{2}=-\infty, [/mm] also ist [mm] Y_{n}=\bruch{1}{n}\summe_{i=1}^{n}ln(X_{i}) \to -\infty [/mm] fast sicher
und damit
[mm] \limes_{n\rightarrow\infty}(X_1\cdot\ldots\cdot X_n )^{\frac{1}{n}} $=\limes_{n\rightarrow\infty}$ e^{Y_n} [/mm] = 0 fast sicher.

Gruß Septime

Bezug
                                        
Bezug
Fast sichere Konvergenz: Antwort
Status: (Antwort) fertig Status 
Datum: 15:27 So 19.06.2016
Autor: Gonozal_IX

Hiho,

>  und mit dem starken Satz der großen Zahlen folgt

warum darfst du das überhaupt anwenden?

> [mm]\bruch{1}{n}\summe_{i=1}^{n}ln(X_{i}) \to E(ln(X_{1}))[/mm] fast sicher.

[ok]

>  Außerdem ist [mm]X_{1}[/mm] gleichverteilt

[ok]

> daher gilt für [mm]b\in(0,1][/mm]
>  
> [mm]E(ln(X_{1}))=\limes_{b\rightarrow0}\bruch{ln(1)+ln(b)}{2}=-\infty,[/mm]

[notok]
Das tut ja fast weh beim Lesen…
Wie berechnet sich der Erwartungswert von f(X) für eine meßbare Funktion f, wenn man die Dichte von X kennt?

Gruß,
Gono

Bezug
                                                
Bezug
Fast sichere Konvergenz: Frage (beantwortet)
Status: (Frage) beantwortet Status 
Datum: 05:45 Mo 20.06.2016
Autor: Septime

Ich hoffe jetzt stimmt alles:

Alle [mm] X_{i} [/mm] sind u.i.v. Zufallsgrößen und die Logarithmusfunktion ist messbar, also ist [mm] (ln(X_{n})) [/mm] ebenfalls eine Folge u.i.v Zufallsgrößen.
Weiterhin gilt wegen Messbarkeit von der Logarithmusfunktion, mit der Gleichverteilung der [mm] X_{i} [/mm] und mit L'Hospital für i=1,...,n
[mm] E(ln(X_{i}))=\integral_{-\infty}^{\infty}{ln(x)f_{X_{i}}(x) dx}=\integral_{-\infty}^{\infty}{ln(x)\gamma_{[0,1]}(x) dx}=\limes_{b\rightarrow0}\integral_{b}^{1}{ln(x) dx}=-1\in\IR [/mm] und damit sind alle Voraussetzungen für den starken Satz der großen Zahlen erfüllt und es gilt
$ [mm] \limes_{n\rightarrow\infty}(X_1\cdot\ldots\cdot X_n )^{\frac{1}{n}} [/mm] $=$ [mm] =\limes_{n\rightarrow\infty} [/mm] $$ [mm] e^{Y_n} $=\bruch{1}{e} [/mm] fast sicher.

Gruß Septime

Bezug
                                                        
Bezug
Fast sichere Konvergenz: Antwort
Status: (Antwort) fertig Status 
Datum: 08:00 Mo 20.06.2016
Autor: Gonozal_IX

Hiho,

> Alle [mm]X_{i}[/mm] sind u.i.v. Zufallsgrößen und die
> Logarithmusfunktion ist messbar, also ist [mm](ln(X_{n}))[/mm]
> ebenfalls eine Folge u.i.v Zufallsgrößen.
>  Weiterhin gilt wegen Messbarkeit von der
> Logarithmusfunktion, mit der Gleichverteilung der [mm]X_{i}[/mm] und
> mit L'Hospital für i=1,...,n
> [mm]E(ln(X_{i}))=\integral_{-\infty}^{\infty}{ln(x)f_{X_{i}}(x) dx}=\integral_{-\infty}^{\infty}{ln(x)\gamma_{[0,1]}(x) dx}=\limes_{b\rightarrow0}\integral_{b}^{1}{ln(x) dx}=-1\in\IR[/mm]
> und damit sind alle Voraussetzungen für den starken Satz der großen Zahlen erfüllt

Kommt drauf an, was ihr für Voraussetzungen definiert habt. Normalerweise setzt man  noch [mm] $\ln(X_i) \in L^2$ [/mm] voraus (auch wenn man das eigentlich nicht braucht…). Solltet ihr das so definiert haben, musst du das natürlich auch prüfen.

> und es gilt  [mm]\limes_{n\rightarrow\infty}(X_1\cdot\ldots\cdot X_n )^{\frac{1}{n}} [/mm]=[mm] =\limes_{n\rightarrow\infty}[/mm][mm] e^{Y_n}[/mm][mm] =\bruch{1}{e}[/mm] fast sicher.

Sofern du die anderen Schritte von vorher mit aufnimmst, ist das ok. So wäre mir das als Korrektor zu kurz.

Gruß,
Gono

Bezug
                                                                
Bezug
Fast sichere Konvergenz: Mitteilung
Status: (Mitteilung) Reaktion unnötig Status 
Datum: 09:21 Mo 20.06.2016
Autor: Septime

Alles klar! Danke, du hast mir sehr weitergeholfen!

Gruß Septime

Bezug
Ansicht: [ geschachtelt ] | ^ Forum "Wahrscheinlichkeitstheorie"  | ^^ Alle Foren  | ^ Forenbaum  | Materialien


^ Seitenanfang ^
www.unimatheforum.de
[ Startseite | Forum | Wissen | Kurse | Mitglieder | Team | Impressum ]